ORTHOPEDIC MCQS OB 20 SHOULDER AND ELBOW4

ORTHOPEDIC MCQS OB 20 SHOULDER AND ELBOW4

 
 
101) Which of the following statements most accurately describes the anatomy and kinematics of the elbow medial ulnar collateral ligament (UCL)? 
1. Anterior band of the anterior bundle exhibits an isometric strain pattern through elbow range of motion (ROM) 
2. Anterior band of the anterior bundle exhibits an isokinetic strain pattern through elbow ROM 
3. Posterior band of the anterior bundle exhibits an isometric strain pattern through elbow ROM 
4. Posterior band of the anterior bundle exhibits an isokinetic strain pattern through elbow ROM
5. Anterior band of the posterior bundle exhibits an isometric strain pattern through elbow ROM 
Corrent answer: 1 
The anterior band of the anterior bundle of the elbow medial UCL exhibits an isometric strain pattern through elbow ROM. 
The elbow medial UCL is a ligamentous complex consisting of the anterior bundle (anterior oblique ligament), the posterior bundle (posterior oblique ligament), and the transverse ligament. The anterior bundle provides the major contribution to valgus stability of the elbow. The anterior bundle is composed of an anterior band, posterior band, and oblique band. The anterior band of the anterior bundle exhibits an isometric strain pattern throughout elbow ROM, while the posterior band of the anterior bundle tightens during elbow flexion.  
Jackson et al. performed a cadaveric study to examine the functional characteristics of the anterior and posterior bands of the anterior bundle of the UCL. They reported that the posterior band demonstrated a linear increase in strain with increasing degrees of elbow flexion while the anterior band demonstrated isometric strain throughout ROM. They concluded that since both the anterior and posterior bands show similar strain in a load to failure model, it is the insertion point, not the intrinsic differences of the bands, which determines the function of the anterior bundle of the UCL.  
Callaway et al. performed a cadaveric study to determine the role of the medial collateral ligament under valgus loading. They demonstrated that the medial UCL was composed of anterior, posterior, and transverse bundles; the anterior bundle further composed of anterior and posterior bands that tightened in reciprocal fashion as the elbow was flexed and extended. They concluded that the anterior bundle was the major restraint to valgus instability and as such, reconstructive procedures should focus on anatomical reproduction of the anterior bundle. 
Illustration A depicts the medial ulnar collateral ligamentous complex.  
Incorrect Answers:  
Answer 2: The anterior band of the anterior bundle exhibits an isometric, not isokinetic strain pattern. 
Answers 3 & 4: The posterior band of the anterior bundle undergoes increase in strain and tension during elbow flexion. 
Answer 5: The posterior bundle of the medial UCL does not subdivide into bands like the anterior bundle.
 
102) A minor league baseball player presents with deteriorating pitch velocity and control in addition to worsening elbow soreness. His MRI is shown in Figure A and based on this he decides to proceed with surgery. Which of the following medial ulnar collateral reconstruction techniques would give this athlete the best chance to return to sport?
1. Modified Jobe technique and ulnar nerve submuscular transposition 2. Classic Jobe technique and ulnar nerve in situ release 
3. Classic Jobe technique and ulnar nerve transposition 
4. Docking technique and ulnar nerve in situ release 
5. Modified Jobe technique and medial epicondylectomy 
Corrent answer: 4 
The docking technique for medial ulnar collateral ligament (MUCL) reconstruction with a muscle splitting approach has been shown in multiple studies to have the highest rate of return to sport. In situ ulnar nerve release is most commonly performed in the absence of pre-operative neuropathy.  
The medial ulnar collateral ligament (MUCL) is the primary restraint to valgus across the elbow, with the anterior bundle contributing the most to stability. Multiple different reconstruction techniques have been developed, yet all are aimed at reconstructing the anterior bundle of the MUCL using a tendon graft.  
Watson et al. reviewed MUCL reconstruction techniques among overhead throwing athletes. They found the highest return to play rate with the docking technique (90%) compared to the classic Jobe (67%) and modified Jobe technique (77%). However, this difference may be confounded by the adoption of muscle splitting techniques, fewer obligatory ulnar nerve transpositions, and advances in rehabilitation. 
Vitale and Ahmad also reviewed MUCL reconstruction techniques. They found avoiding obligatory ulnar nerve transposition had a significantly lower rate of post-operative neurapraxia (3% vs 8%) and better outcomes (90% excellent 
vs 76% excellent results). This study highlights the positive effects of research and refinement of MUCL reconstruction techniques.  
Arner et al. compared cohorts who underwent either the docking technique or the modified Jobe technique at a single institution. They reported similar KJOC scores, Conway scores, and rates complications and return to sport between both cohorts. These findings may show there to be fewer differences between these techniques when interchangeably performed by a single, experienced surgeon. 
Figure A is T2 coronal MR sequence showing a torn MUCL. Illustration A demonstrates the docking technique. 
Incorrect Answers:  
Answer 1: Routine ulnar nerve transposition is not advised without pre operative neuropathy.
Answers 2 and 3: The classic Jobe technique has fallen out of favor and is done sparingly.  
Answer 5: Medial epicondylectomy should be avoided in throwing athletes.
 
103) A 35-year-old athlete presents with chronic left shoulder pain with worsening external rotation weakness when his elbow is at his side. He has full external rotation when his arm is elevated. His clinical picture is depicted in Figure A. If an MRI is obtained, which of the following images (Figures B-F) would most likely be found?
1. Figure B 
2. Figure C 
3. Figure D 
4. Figure E 
5. Figure F 
Corrent answer: 2 
This patient is presenting with isolated infraspinatus atrophy and weakness likely due to compression of the suprascapular nerve at the spinoglenoid notch. MRI would likely reveal a spinoglenoid cyst with fatty atrophy of the
infraspinatus.  
Suprascapular neuropathy can result from compression of the suprascapular nerve at the suprascapular notch or the spinoglenoid notch. When compression occurs at the suprascapular notch, both the supraspinatus and infraspinatus are affected. When compression occurs at the spinoglenoid notch (likely from a labral tear), only the infraspinatus is affected, with the supraspinatus spared. Isolated infraspinatus palsy would manifest as weakness in arm external rotation with elbow at side. 
Piaseki et al. published a review paper on suprascapular neuropathy. They reported that there are several potential causes of nerve entrapment along the path of the suprascapular nerve, including the suprascapular and spinoglenoid notches, where nerve excursion is limited by bony and ligamentous constraints. Furthermore, compression may be caused by ganglion cysts and soft-tissue masses about the shoulder. They recommended initial treatment be primarily nonsurgical with activity modification and physical therapy in the absence of clear nerve compression, which would warrant surgical intervention. 
Contemori et al. performed a study to evaluate the EMG activity patterns and relationships of relevant shoulder girdle muscles in professional volleyball players with isolated infraspinatus atrophy (IIA). They reported that athletes with IIA had significantly higher deltoid and trapezius muscles activity and lower serratus anterior activity compared with the contralateral shoulder and healthy athletes. They concluded that the shoulder muscle impairments seen in IIA may compromise the optimal scapulohumeral rhythm and function, and increases the risk of acute and overuse shoulder injuries. 
Figure A is a clinical picture depicting isolated atrophy of the infraspinatus muscle. Figure B depicts a partial thickness supraspinatus tear. Figure C depicts a spinoglenoid cyst with mild fatty atrophy of the infraspinatus muscle. Figure D depicts supraspinatus and infraspinatus atrophy. Figure E depicts a Hill-Sachs lesion with posterior dislocation of the humeral head. Figure F depicts atrophy of the teres minor muscle. Illustration A depicts the anatomy of the suprascapular nerve and potential sites of compression. 
Incorrect Answers: 
Answer 1: A partial thickness supraspinatus tear would not present with atrophy and weakness of the infraspinatus. 
Answer 3: Combined supraspinatus and infraspinatus atrophy would more likely be caused by compression at the suprascapular notch, and would result in an inability to externally rotate with the arm in forward elevation.
Answer 4: A Hill-Sachs lesion with posterior humeral subluxation would not result in this patient's clinical exam. 
Answer 5: Isolated teres minor atrophy is more likely to be caused by compression of the axillary nerve in the quadrangular space.  
 
104) A 47-year-old woman presents with concerns of chronic right shoulder pain and stiffness without antecedent trauma. She has significantly limited right shoulder active and passive range of motion (ROM) in all planes but full left shoulder active and passive ROM. Her MRI is depicted in Figure A. What is the pathophysiology of her diagnosis and what other findings would most likely be observed?
1. Isolated posterior capsular tightness; Hemoglobin A1C 11.7% 2. Isolated posterior capsular tightness; TSH 15 mU/L 
3. Fibroblastic proliferation of joint capsule; Hemoglobin A1C 11.7% 4. Decreased blood supply to humeral head leading to bony matrix cell death; TSH 15 mU/L 
5. Chronic degenerative tear of shoulder-stabilizing tendons; Hemoglobin A1C 11.7% 
Corrent answer: 3 
The clinical presentation is consistent with shoulder adhesive capsulitis (frozen shoulder), which occurs by fibroblastic proliferation of the joint capsule and is associated with endocrine abnormalities including diabetes and hypothyroidism.  
Adhesive capsulitis results in painful, gradual unexplained loss of active and passive shoulder ROM. It is thought to involve a fibroblastic proliferative process resulting in joint capsular contracture. Risk factors include diabetes, thyroid disorders, history of myocardial infarcation, Crohn's disease, and prolonged immobilization. While plain radiographs will reveal no osseous abnormalities, MRI demonstrates loss of the normal axillary recess. Initial treatment includes gentle physical therapy stretching, with surgical intervention reserved for failure of nonoperative management for at least 6 months. However, resolution of symptoms often occurs without significant intervention. 
Schiefer et al. investigated the prevalence of hypothyroidism in patients with adhesive capsulitis in a case-control study. They reported that the prevalence of hypothyroidism was significantly higher in the adhesive capsulitis cohort, and mean serum TSH levels were higher in patients with bilateral adhesive capsulitis compared to those with unilateral disease. They concluded that higher serum TSH levels were associated with bilateral and severe cases of adhesive capsulitis and recommended evaluation of thyroid function during assessment of patients with frozen shoulder. 
Chan et al. published a study to investigate the correlation between cumulative HbA1c levels in diabetic patients and the prevalence of frozen shoulder. They reported that a cumulative HbA1c was positively associated with adhesive capsulitis, with a significant effect size. They concluded that patients with worse blood sugar control over a longer period are at an increased risk of developing adhesive capsulitis. 
Figure A depicts right shoulder loss of the normal axillary recess which is indicative of capsular contracture. Illustration A depicts a normal axillary
recess in comparison to an absent axillary recess seen in adhesive capsulitis.  
Incorrect Answers:  
Answers 1 & 2: Isolated posterior capsular tightness is seen in glenohumeral internal rotation deficit, not adhesive capsulitis.  
Answer 4: Decreased blood supply to the humeral head is seen with avascular necrosis, which is not seen on this patient's MRI. 
Answer 5: A chronic degenerative rotator cuff tear is not seen on this patient's MRI, and typically would result in loss of active but not passive motion indicating weakness rather than stiffness. 
 
105) A 73-year-old man with a history of a left shoulder hemiarthroplasty 10 years ago sustains a fall and presents the the emergency room with acute pain and weakness in his left shoulder. Injury radiographs are shown in Figure A. He undergoes successful closed reduction of the left shoulder. On routine follow up 3 months later, he is still unable to elevate his arm beyond 30 degrees despite rigorous physical therapy. New radiographs depict anterosuperior escape. What is the structure likely injured and what is the treatment that would result in the most predictable outcome?
1. Rotator cuff; Isolated rotator cuff repair 
2. Rotator cuff; Rotator cuff repair and conversion to anatomic total shoulder arthroplasty 
3. Rotator cuff; Conversion to a reverse total shoulder arthroplasty 4. Labrum; Isolated labral repair 
5. Labrum; Conversion to anatomic total shoulder arthroplasty Corrent answer: 3 
Rotator cuff injuries are associated with traumatic instability episodes after shoulder hemiarthroplasty. Conversion to a reverse total shoulder arthroplasty (RTSA) would offer the best and most predictable outcomes given the patient's age and integrity of the rotator cuff musculature.  
RTSA is a viable treatment option for patients with failed shoulder arthroplasty with anterosuperior escape because it allows for improved arm elevation and abduction without subluxation in the setting of nonfunctional rotator cuff muscles. In patients with anterosuperior escape, hemiarthroplasties have traditionally not performed well and poor results have been reported in patients who undergo rotator cuff repair (RCR) after shoulder arthroplasty.  
Kany et al. performed a study to investigate the predictors of postoperative
instability after shoulder arthroplasty. They reported rotator cuff tears in the majority of instability episodes and that patients who were converted to a RTSA had significantly better outcomes compared to those who underwent other procedures. They concluded that soft tissue deficiency was the main cause of shoulder prosthesis instability and that conversion to RTSA led to predictable and reliable results. 
Hattrup et al. performed a retrospective study to assess the results of RCR after shoulder replacement. They reported that RCR was unsuccessful in the majority of patients (77%) with limitations in range of motion as a result. They recommended that, at the time of index arthroplasty, a careful attempt be made to balance the soft tissues and securely repair the rotator cuff since subsequent attempts to repair the rotator cuff are prone to failure. 
Abboud et al. published a review article on soft-tissue management in revision total shoulder arthroplasty (TSA). They reported that unstable arthroplasty can be the result of asymmetric soft-tissue balancing and deficiencies that result in 
loss of the rotator cuff force couples and ultimately instability during glenohumeral motion. They propose an organized approach in diagnosis and management of the stiff or unstable TSA.  
Figure A depicts a dislocated shoulder hemiarthroplasty. 
Incorrect Answers: 
Answers 1 & 2: Poor results have been reported in patients that underwent rotator cuff repair after shoulder arthroplasty. Furthermore, conversion to an anatomic TSA would not address the deficient rotator cuff force couples.  Answers 4 & 5: Rotator cuff injuries are more likely than labral injuries after instability events in this age group. 
 
106) What is the function of the structure indicated by the arrow in Figure A, visualized through the posterior portal in the lateral decubitus position?
1. Resists inferior translation of the humerus at 0 degrees abduction 2. Resists anterior and posterior translation of the humerus at 45 degrees abduction 
3. Resists posterior translation of the humerus at 90 degrees flexion and internal rotation 
4. Resists posterior translation of the humerus with shoulder in flexion, adduction, and internal rotation 
5. Functions to internally rotate the humerus 
Corrent answer: 2 
The arrow in Figure A depicts the middle glenohumeral ligament (MGHL). The MGHL resists anterior and posterior translation of the humerus at the midrange of abduction, at 45 degrees.  
The MGHL is a static glenohumeral ligament, originating from the anterior glenoid labrum, running obliquely across the subscapularis, and inserting just medial to the humeral lesser tuberosity. There are several normal variants of the MGHL, including the cord-like MGHL in the Buford complex. Thus, it is important to recognize it and differentiate it from the superior glenohumeral ligament (SGHL), inferior glenohumeral ligament (IGHL), and subscapularis during arthroscopic procedures.  
Wang et al. performed a basic science study on the pathomechanics of acquired shoulder instability. They highlighted the anatomy of the static glenohumeral ligaments, reporting them to be complex and variable, with their function significantly dependent on the position of the humerus with respect to the glenoid.
Itoigawa et al. published a review on the anatomy of the capsulolabral complex and rotator interval related to glenohumeral instability. They reported that the MGHL was the most variable ligament among the glenohumeral ligaments. They emphasized that knowledge of the detailed anatomy and anatomical variations is essential for the orthopedic surgeon in understanding the pathology, which results in accurate diagnosis of instability, and ultimately correct treatment options. 
Figure A depicts the glenohumeral joint, viewed from the posterior arthroscopic portal in a lateral decubitus position. Illustration A depicts labeled anatomic structures visualized during shoulder arthroscopy.  
Incorrect Answers: 
Answer 1: This describes the function of the SGHL. 
Answer 3: This describes the function of IGHL. 
Answer 4: This describes the function of the coracohumeral ligament. Answer 5: This describes the function of the subscapularis tendon. 
 
107) A 47-year-old man presents to your clinic with chronic persistent deep-seated right shoulder pain. He is able to perform most daily activities but experiences sharp shooting pain within the anterior aspect of the shoulder whenever he thows a baseball with his son. His primary care physician referred him to physical therapy first, which resulted in limited improvement in his symptoms, but he remains
dissatisfied. Examination reveals symmetric rotator cuff strength and no instability. Speed's and O'brien's tests are positive. His MRI is shown in Figure A. Which of the following is correct with respect to the pathologic structure? 
1. Anchors the biceps tendon in a predominantly anterior position 2. Supplied by a single arterial source 
3. The anterior superior portion is most susceptible to vascular insufficiency 4. Degeneration usually leads to cyst formation within the suprascapular notch 5. Repair should be accompanied by post-operative limitation in abduction and internal rotation 
Corrent answer: 3 
The patient has a superior labral (SLAP) tear as evident by the clinical presentation, physical exam, and imaging. The anterior superior portion of the labrum has the most tenuous blood supply and as a result, is most susceptible to degenerative tears with a reduced healing capacity.  
SLAP tears are one of the most common causes of shoulder pain and weakness in the throwing population. Repetitive overhead activities often result in tightness of the posterior capsule and ligamentous structures of the glenohumeral joint which shifts the glenohumeral contact point and increases the shear force on the superior labrum, potentiating SLAP tears. This is the so called "peel-back" lesion, where the humeral head translation causes the labrum to peel back from its attachment site on the glenoid. The labrum also
anchors the biceps tendon, which is the reason that proximal bicipital tendon pathology can present with symptoms and physical exam findings similar to a labral tear. 
Snyder et al. first presented this specific pattern of labral tears in a study of 700 shoulders. The authors noted that the superior labral tear began posteriorly and extended anteriorly, stopping before or at the mid-glenoid notch and including the "anchor" of the biceps tendon to the labrum. They labeled this lesion the "SLAP." They also reported that the most common mechanism of injury was a compression force to the shoulder, usually as a result of a fall onto an outstretched arm, with the shoulder positioned in abduction and slight forward flexion at the time of the impact. 
Burkhart et al. reviewed the pathoanatomy of the SLAP tear and discussed common physical exam findings. They noted that the SLAP tear often produces a deep shoulder pain and is the most common cause of a "dead" arm. The authors also note that the two most specific tests for an anterior SLAP tear are the Speed's test and the O'Brien's test. They go on to describe arthroscopic techniques for the repair of such lesions.  
Figure A is a coronal T2 MRI revealing a fluid signal between the superior labrum and glenoid indicative of a SLAP tear.  
Incorrect Answers: 
Answer 1: The biceps tendon is most commonly anchored in a position posterior to 12 o'clock.  
Answer 2: The labrum receives its blood supply from the capsule and periosteal vessels, which are derived from the suprascapular artery, the circumflex scapular artery, and the posterior humeral circumflex artery.  Answer 4: SLAP tears are most often associated with cysts within the spinoglenoid and NOT suprascapular notch. 
Answer 5: The post-operative protocol following a SLAP repair includes limitation of abduction and EXTERNAL rotation, as this places maximal tension on the repair. 
 
108) Figures A-C are the CT images of a 20-year-old basketball player who is referred to your office due to recurrent shoulder dislocations. The humeral-sided lesion is determined to be "off-track." His initial dislocation occurred 3-years-ago during a basketball game. He has since had 9 recurrent episodes and is able to self-reduce the shoulder. He has trialed two courses of formal phyical therapy without
improvement. The decision is made to proceed with surgical treatment. At the first postoperative visit, he complains of lateral forearm numbness and examination reveals forearm supination weakness. Which neuropraxia was encountered and which procedure was indicated?
1. Musculocutaneous & Arthroscopic Bankart Repair 
2. Musculocutaneous & Latarjet 
3. Median & Latarjet 
4. Radial & Open Bankart Repair 
5. Radial & Latarjet 
Corrent answer: 2 
This patient has recurrent shoulder dislocations with significant glenoid bone loss on CT imaging indicating the need for a bony augmentation procedure (ie. Latarjet). His postoperative examination reveals a musculocutaneous neuropraxia.  
This patient sustained an initial traumatic shoulder dislocation at the age of 17-years. The recurrence rate for patients <20-years is >90%. In addition to the recurrent dislocations, this patient's CT imaging demonstrates significant glenoid wear. With glenoid wear exceeding 20%, the ability of a soft tissue only procedure (ie. Bankart repair) is unlikely to restore adequate stability of the shoulder. In these patients, a bony augment is utilized to improve stability. The Latarjet triple effect includes bony restraint by increasing the glenoid track, a sling effect secondary to the conjoined tendon resting on top of subscapularis, and an anterior restraint from capsule reconstruction via the CA ligament.  
Shin et al. reviewed the critical value of anterior glenoid bone loss that led to surgical failure in patients with anterior shoulder instability. They report that
the optimal critical value of glenoid bone loss was 17.3%. They concluded that anterior glenoid bone loss exceeding 17.3% should be considered as the critical amount of bone loss that may result in recurrent glenohumeral instability after arthroscopic Bankart repair.  
Streubel et al. review the recurrence of glenohumeral joint dislocations in young male athletes competing in contact sports. They report that diagnostic imaging is critical in assessing bone loss of the glenoid or humeral head. They concluded that in the setting of anterior glenoid bone loss >20% of the articular surface, iliac crest bone grafting or coracoid transfer via the Bristow or Latarjet procedures has demonstrated satisfactory outcomes.  
Gupta et al. reviewed the complications following the Latarjet procedure. In their systematic review, they report a 1.4% rate of neurovascular injury across open and arthroscopic techniques; 11 musculocutaneous nerve injuries, of which 2 were either partial or temporary, while 9 were either partial permanent or complete injuries resulting in nerve deficit. They concluded that a musculocutaneous nerve palsy remains a well-documented albeit rare complication of this procedure. 
Clavert et al. reviewed the anatomical relationships between the musculocutaneous nerve and the coracobrachialis in patients undergoing coracoid transfer. They found that lesion of the musculocutaneous nerve is secondary to lengthening of the nerve and modification of the penetration angle into the coracobrachialis. They concluded that some motor nerve destined to the coracobrachialis might be damaged during the proximal medial release of the muscle after the detachment of the pectoralis minor muscle.  
Delaney et al. utilized intraoperative neuromonitoring to define the stages of the Latarjet procedure during which the nerves are at greatest risk. They reported that 76.5% of cases had nerve alert episodes, occurring most commonly during glenoid exposure and graft insertion. They noted that 20.6% of cases had a clinically detectable nerve deficit postoperatively, with all cases involving the axillary nerve and all resolving completely from 28-165 days postoperatively. They concluded that nerves, in particular the axillary and musculocutaneous nerves, are at risk during the Latarjet procedure, especially during glenoid exposure and graft insertion.  
Figure A is a CT image demonstrating significant glenoid wear. Figures B&C are the axial and coronal CT images revealing a Hill-Sachs lesion of the humeral head. 
Incorrect Answers:
Answers 1&4: A soft tissue only procedure (ie. Bankart repair) would not adequately address instability in the setting of significant glenoid bone loss and an off-track Hill-Sachs lesion 
Answers 3&5: This patient's postoperative examination is concerning for a musculocutaneous neuropraxia as evident by his lateral forearm numbness (lateral antebrachial cutaneous nerve is a branch of the musculocutaneous nerve) and supination weakness (biceps is primarily a supinator) 
 
109) A 17-year-old male injures his right elbow during an athletic contest. In the office, the surgeon externally rotates the humerus and supinates the forearm while applying a valgus stress to the elbow on the injured upper extremity. The patient feels apprehension as his elbow is ranged between 70 and 120 degrees. Which of the following structures is most likely injured? 
1. Medial ulnar collateral ligament 
2. Lateral ulnar collateral ligament 
3. Annular ligament 
4. Anterior capsule 
5. Triceps 
Corrent answer: 1 
The described moving valgus stress test is used in the diagnosis of medial ulnar collateral ligament (MUCL) injury of the elbow.  
Attenuation or complete rupture of the MUCL leads to valgus instability. This is common in overhead athletes, particularly baseball pitchers. During the late cocking/early acceleration phase of throwing, increased torque is placed upon the medial aspect of the elbow. Poor throwing mechanics also places an increased valgus load on the medial aspect of the elbow and can precipitate injury. The moving valgus stress test is a key physical exam maneuver and is reported to be 100% sensitive and 75% specific for MUCL injury. Competitive athletes may elect to undergo repair or reconstruction while others may be treated non-operatively.  
Savoie and O'Brien provide a comprehensive review of chronic medical elbow instability. They note that typically, MUCL injuries heal without operative management, especially in patients who are not high-functioning. Physical therapy may aid in the cases of symptomatic persistent instability. However the authors discuss that repair or reconstruction should be considered in throwing athletes, wrestlers, and other highly active athletes, and that
outcomes of surgery are good in as many as 84-94% of cases.  
O'Driscoll et al. reviewed 21 cases of patients who underwent surgery for medial elbow instability with a positive preoperative moving valgus stress test. The test proved to be 100% sensitive and 75% specific. Based on surgical findings the authors report that the moving valgus stress test is a highly accurate physical exam technique.  
Illustration A demonstrates the moving valgus stress test.  
Incorrect Answers: 
Answer 2: An LUCL injury can be elucidated with the posterolateral rotatory instability test. 
Answer 3: The annular ligament may be injured in a terrible triad injury. Answer 4: The chair test is positive when the patient has apprehension or dislocation on the terminal extension of the supinated forearm when rising from a seated position. This is indicative of PLRI. 
Answer 5: The Pushup sign is apprehension or inability to push up off of the floor when the forearm supinated. This is indicative of PLRI. 
 
110) A competitive powerlifter presents to your office for a few months of deep, intermittent shoulder pain. It is worse while performing the bench press but doesn't bother him with overhead activities or while carrying items at his side. Exam shows full painless active shoulder motion, normal rotator cuff strength, with no muscular atrophy or asymmetry. His pain is reproduced while doing a pushup. He is referred for an MRI and course of physical therapy. What pathology do you expect the MRI to show?
1. Anteroinferior capsulolabral separation 
2. Tearing of the clavicular portion of the pectorals major tendon 3. Posterosuperior humeral head bone defect 
4. Posteroinferior capsulolabral separation 
5. Superior labral tear involving the biceps anchor 
Corrent answer: 4 
This presentation is classic for posterior labral tears. MRI would show posteroinferior capsulolabral separation or tear.  
Posterior labral tears are uncommon in the general population. Patients most commonly present with vague shoulder pain as opposed to overt instability. Occupations or activities that involve axial loading of the shoulder while forward flexed and adducted are most often implicated, as with weight lifters or football linemen. Exam findings may be subtle but the Kim test, jerk test, and/or load-and-shift tests have all been demonstrated to indicate posterior labral tears. Physical therapy is often successful initial treatment, especially in cases of generalized ligamentous laxity.  
Provencher et al. reviewed posterior shoulder instability. Static posterior stability relies on the capsuloligamentous structures and bony restraints, and they noted the posterior-inferior glenohumeral ligament is thinner and weaker than the anterior counterpart. This means the dynamic stabilizing effect of the subscapularis is critical and should be evaluated in all cases of posterior instability.  
Millett et al. reviewed recurrent posterior shoulder instability. They advocate careful scrutiny of imaging studies, as successful surgical management relies on addressing all pathologic structures. For isolated labral tears, the arthroscopic repair will suffice, but for labral tears with glenoid retroversion, an open-wedge osteotomy will be necessary.  
Illustration A is an axial MRI with a posterior labral tear. Illustration B is an axial MRI showing an anterior Bankart labral tear with concomitant Hill-Sachs lesion. Illustration C is a coronal T2 MRI showing a HAGL lesion, while Illustration D shows a SLAP tear.  
Incorrect Answers:  
Answers 1 and 3- These describe a Bankart lesion and HIll-Sachs lesion, respectively. These occur following anterior shoulder instability events, and exam findings would likely show apprehension with abduction and external rotation. 
Answer 2- A pec major tear would show loss of strength with resisted
adduction and internal rotation, and likely bruising and loss of the anterior axillary fold contour.  
Answer 5- SLAP lesions are symptomatic most commonly during overhead activities. Throwing athletes may describe dead-arm symptoms as well.
 
111) A 72-year-old male undergoes a right anatomic total shoulder arthroplasty through a standard deltopectoral approach. There were no complications during the procedure. Which movement should be avoided during the immediate post-operative period?
1. Passive internal rotation to the chest 
2. Passive shoulder forward elevation to 90° 
3. Pendulum exercises 
4. Active elbow flexion to 100° 
5. Passive shoulder external rotation to 60° 
Corrent answer: 5 
The patient underwent a deltopectoral approach for an anatomic total shoulder arthroplasty, which requires a subscapularis tenotomy, peel, or lesser tuberosity osteotomy. Regardless of how the subscapularis is managed, in the postoperative period the subscapularis repair needs to be protected. As such, passive shoulder external rotation beyond 30° should be avoided in order to protect the subscapularis tendon repair.  
Total shoulder arthroplasty is an effective treatment for the degenerative joint disease of the glenohumeral joint but is dependent on a competent rotator cuff for successful outcomes. During the standard deltopectoral approach, the subscapularis tendon is incised near the insertion to the lesser tuberosity in order to gain access to the glenohumeral joint. In the immediate postoperative period, shoulder external rotation should be limited to protect the subscapularis tendon repair.  
Boardman et al. performed a study of 77 patients and 81 shoulders treated with an anatomic total shoulder arthroplasty and underwent a graduated rehabilitation program. They reported that 70% of patients maintained their intraoperative elevation and 90% maintained their intraoperative external rotation with the prescribed rehabilitation program detailed in illustration A. The authors concluded that their rehabilitation program is effective at maintaining shoulder motion after shoulder arthroplasty with better motion and tendon healing expected for patients with primary osteoarthritis.  
Armstrong et al. performed a retrospective EMG study of 30 patients that underwent an anatomic total shoulder arthroplasty. They reported normal EMG findings in 15 patients and chronic denervation and reinnervation in the remaining 15 patients which affected the subscapularis in 30%, the infraspinatus in 27%, the supraspinatus in 20%, the teres minor 20%, and the rhomboids in 13% of patients. The authors theorized surgical exposure, traction, and interscalene nerve block may contribute to these EMG findings.  
Illustration A is the graduated rehabilitation program that was prescribed by Boardman et al. to patients undergoing anatomic total shoulder arthroplasty.  
Incorrect answers:
Answer 1: Passive internal rotation to the chest would not affect the integrity of the subscapularis repair.  
Answer 2: Passive forward shoulder elevation to 90° would not affect the integrity of the subscapularis repair and helps prevent excessive shoulder stiffness from occurring.  
Answer 3: Shoulder pendulum exercises place no added stress to the subscapularis repair and allow for a safe early range of motion.  Answer 4: Active elbow flexion to 100° would have no bearing on the subscapularis repair. 
 
112) Which technical modification can minimize the complication depicted in figure A?
1. Horizontal humeral cut 
2. Superior placement of the baseplate 
3. Inferior placement of the baseplate 
4. Use of a smaller glenosphere 
5. Use of a retentive polyethylene liner 
Corrent answer: 3 
Figure A depicts scapular notching which can be minimized by the inferior placement of the glenoid baseplate.  
Scapular notching in reverse total shoulder arthroplasty is a common finding and usually presents with 1-2 years from surgery. Inferior scapular notching is the most common form and is usually the result of the superior placement of the glenoid component and with inadequate inferior tilt. Rarely does scapular notching necessitate revision surgery, but has been implicated with decreased abduction and flexion ROM, post-operative strength, and patient-reported outcomes compared to patients without notching.  
Kontaxis and Johnson performed a biomechanical study using 3-dimensional modeling software. They reported a 42% increase in the moment arm of the deltoid, which adequately compensated for dysfunctional rotator cuff muscles. but identified the impingement of the humeral prosthesis with the bone of the scapular body. They concluded that reverse prosthesis designs are able to provide an improved biomechanical function to the glenohumeral joint in the setting of rotator cuff deficiency.  
Gutiérrez et al. performed a biomechanical study of reverse total shoulder arthroplasty prosthesis. They reported that a 15° inferior tilt resulted in the most uniform compressive forces, the least amount of tensile forces, and the least amount of micromotion on the glenoid component. They concluded inferiorly tilting of the glenoid component will result in decreased mechanical failure.  
Simovitch et al. performed a retrospective review of risk factors and clinical implications of scapular notching following reverse total shoulder arthroplasty. The authors found that inferior placement of the glenoid prosthesis with smaller scapular neck angle decreased the incidence of inferior spurs and was associated with improved patient-reported outcomes. The authors concluded that optimal glenosphere placement reduces inferior scapular notching and improves patient-reported outcomes. 
Figure A is an AP radiograph of the right shoulder with reverse total shoulder arthroplasty and evidence of scapular notching on the inferior glenoid. 
Incorrect answers: 
Answer 1: A horizontal humeral neck cut would not decrease the risk of scapular notching.  
Answer 2: The superior placement of the glenoid component would likely result in increased scapular notching.  
Answer 4: Glenosphere size has not been shown to affect scapular notching.  Answer 5: A retentive polyethylene liner would not reduce the risk of scapular notching.  
 
113) A 79-year-old female presents to your clinic with severe left shoulder pain. She has a history of rheumatoid arthritis and is currently on etanercept. She has tried multiple rounds of intra articular steroid injections and these have stopped being effective in managing her pain. Her radiographs are shown in Figures A and B. MRI studies reveal a 2 cm superior complete rotator cuff tear without associated fatty atrophy. Which of the following is true with respect to the surgical management of this patient?
1. Usage of etanercept is associated with decreased functional outcomes after shoulder arthroplasty 
2. Clinical outcomes after Hemiarthroplasty are equivalent to those after total shoulder arthroplasty for this patient's cohort 
3. Reverse total shoulder arthroplasty has shown good clinical outcomes for this patient's cohort 
4. Interpositional arthroplasty and rotator cuff repair has shown good clinical outcomes for this patient's cohort 
5. Patients with inflammatory arthritis have more complications after reverse total shoulder arthroplasty than those with cuff-tear arthropathy 
Corrent answer: 3
The patient has significant arthritis of the glenohumeral joint secondary to rheumatoid arthritis (RA) and has a rotator cuff tear (RCT) on MRI studies. Recent studies have shown that patients with RA and RCTs who undergo a reverse total shoulder arthroplasty (RSA) alone have great clinical outcomes. 
Glenohumeral arthritis secondary to RA had previously been primarily managed with hemiarthroplasty (HA) and total shoulder arthroplasty (TSA). Most studies have shown that TSAs yield better pain relief and improved forward elevation when compared with HA in patients with RA. More recently, the RSA has shown to be an excellent option for patients with RA even in the setting of reparable rotator cuff tears. As such, current studies advocate for either TSA with rotator cuff repair or a RSA for definitive management of such patients.  
Mangold et al. reviewed 91 patients who underwent RSA for inflammatory arthritis with an average follow-up of 4 years. Peri-operative use of steroids, biologics, and methotrexate were reviewed. The authors noted that the two and five-year implant revision-free survival was 99% and patients experienced significant pain relief with a 92% satisfaction rate. The authors noted that the use of prednisone, DMARDs, or biologic medications had no significant impact on outcomes. Shoulder elevation and external rotation improved from 65 and 21 degrees pre-operatively to 138 and 45 degrees post-operatively, respectively. The authors concluded that an RSA is a safe and effective option for the management of inflammatory arthritis even in the setting of prednisone, DMARDs and other biologic medication. 
Woodruff et al. performed a retrospective analysis of 17 TSAs in patients with RA of the glenohumeral joint complicated by rotator cuff dysfunction. Thirteen shoulders were followed up for more than 5 years (mean 87 months). Median Constant-Murley score was 59.0; median scores for general health were 33.40 
and 49.36 for the physical and mental components respectively. The radiographical analysis revealed evidence of lucencies about the humeral component in all cases and about the glenoid component in five cases. The authors concluded that despite the good clinical results, the high incidence of radiographical lucencies is of concern. 
Collins et al. performed a prospective analysis of the clinical, functional, and radiographic outcomes of shoulder reconstruction with HA or TSA for patients with inflammatory arthritis. They looked at 36 patients treated with HA and 25 treated with TSA with 2-year follow-up. All patients showed significant improvement in the VAS pain scores. They noted that forward elevation was significantly better for patients who undergo TSA compared to HA. The authors concluded that patients with inflammatory arthritis treated with TSA or HA can
be expected to have improved comfort, and function with those undergoing TSA having a more significant improvement in ROM than those undergoing HA. 
Cho et al. performed a systematic review to investigate the radiological and clinical outcomes after RSA in patients with RA. They found a total of 7 studies looking at 123 shoulders that met the inclusion criteria at a mean follow-up of 4 years. The mean Constant score and American Shoulder and Elbow Surgeons (ASES) score increased from 18.6 and 27.5 preoperatively to 58.6 and 73.7, respectively, at the final follow-up evaluation. Nine shoulders (7.3%) had one or more revision surgeries. The authors concluded that their systemative review of RSA in RA showed similar short- to mid-term results without higher complication rates as compared to RSA in cuff-tear arthropathy cited previously in literature. 
Figures A and B are the AP and axillary lateral radiographs showing significant glenohumeral arthritis of the left shoulder 
Incorrect Answers: 
Answer 1: Studies have shown that the use of prednisone, DMARDs, or biologic medications had no significant impact on the outcomes of RSA in those with RA. 
Answer 2: Patients who undergo TSA for inflammatory arthritis have better clinical outcomes with improved ROM than those undergoing HA. Answer 4: Interpositional arthroplasty is not indicated for elderly patients with glenohumeral arthritis.  
Answer 5: RSA in RA has shown similar short- to mid-term results without higher complication rates as compared to RSA in cuff tear arthropathy.  
 
114) A 71-year-old female presents with continued right shoulder pain and weakness after sustaining a fall on her right shoulder a few weeks ago. She has trialed physical therapy, which has failed to improve her symptoms. An MRI is obtained, which demonstrates a 2.8 cm supraspinatus tear with minimal retraction and no fatty atrophy. She elects to undergo surgery. When discussing rotator cuff repair with this patient, which of the following statements below is most accurate? 
1. Recurrent tear rate is decreased when repair is performed arthroscopically 2. She will have superior functional outcomes if she undergoes double-row repair compared to single-row
3. She will have a quicker return to function if performed open 4. There should be no difference in the recurrent tear rate if her tear is repaired arthroscopically or open 
5. Her current age is a protective factor for her regardless of the repair method 
Corrent answer: 4 
Of the above statements, the most accurate is that for this patient with a medium-sized rotator cuff tear, there should be no difference in the recurrent tear rate if her tear is repaired arthroscopically or open. 
The objective of rotator cuff repair (RCR) surgery is to relieve pain and restore function, with the ultimate goal of high initial fixation strength, minimization of gap formation, achieving mechanical stability under cyclic loading, and optimizing the biology of the tendon-bone interface until the cuff heals to the bone. While several studies have demonstrated an increased tear recurrence rate with single-row compared to double-row repairs, the literature has failed to show any difference in functional outcomes and recovery after arthroscopic versus standard open RCR. 
Bishop et al. investigated rotator cuff integrity after arthroscopic versus open RCR at 1 year postoperatively. They reported that the American Shoulder and Elbow Surgeons (ASES) and Constant scores significantly improved in both cohorts. They also observed that while there was no difference in cuff integrity for small tears (<3 cm) with either method, for larger tears (>3 cm), the cuff was intact in 62% of the open group versus 24% in the arthroscopic group. They concluded that open and arthroscopic RCRs have similar clinical outcomes and cuff integrity for smaller tears.  
Cole et al. reviewed different suture-anchor repair configurations during RCR. They reported that the single-row RCR is less successful at footprint restoration of the rotator cuff and is most susceptible to gap formation, while double-row RCR has improved load to failure and minimal gap formation. Furthermore, they highlighted that transosseous equivalent repairs exhibited the highest ultimate load and resistance to shear and rotational forces and the lowest gap formation.  
Incorrect Answers: 
Answer 1: The literature has failed to show any difference in functional outcomes and recovery after arthroscopic versus standard open RCR for small and medium cuff tears. 
Answer 2: While double-row RCR exhibits lower tear recurrence rates compared to single-row, functional outcomes are similar. 
Answer 3: There is earlier return to rehabilitation with the arthroscopic RCR as there is no deltoid repair to protect as with the open RCR. 
Answer 5: Her current age of 71-years is a risk factor (not protective) against postoperative cuff integrity. 
 
115) A 26-year-old rugby player undergoes surgery for recurrent instability of his left shoulder. His postoperative radiographs are pictured in Figures A and B. At two week follow up, he is found to have decreased sensation over the lateral shoulder and is unable to actively abduct the shoulder. The injury most likely occurred during ___, due to ___, and the next best step in management is ___.
1. Glenoid exposure; nerve traction; observation 
2. Glenoid exposure; nerve traction; obtain electrodiagnostic studies (EMG/NCV) 
3. Graft insertion; nerve entrapment; obtain electrodiagnostic studies (EMG/NCV)
4. Subscapularis split; nerve traction; observation 
5. Subscapularis split; nerve laceration; immediate exploration Corrent answer: 1 
Neurologic injury during a Latarjet procedure is most commonly due to nerve traction during glenoid exposure or graft insertion and is best managed by close observation for 3-6 weeks. 
The Latarjet procedure is gaining popularity for use in the reconstruction of an anterior glenoid defect associated with recurrent anterior glenohumeral instability. The general surgical technique involves transferring the coracoid process to the anterior glenoid through a split in the subscapularis. Nerves most at risk during this procedure include the axillary and musculocutaneous. Most neurologic injuries following the Latarjet procedure are neuropraxias that resolve spontaneously without intervention. 
Boardman et al. found the vast majority of neurologic injuries occur due to traction or contusion, rarely by direct laceration or nerve entrapment. These injuries are best avoided by careful arm positioning and retractor placement. Due to their predominantly transient nature of these neurologic complications, observation is the treatment of choice. Should nerve recovery not occur within a 3-6 week period, electrodiagnostic testing is recommended to further define the lesion.  
Delaney et al. reported a series of 34 patients who underwent the Latarjet procedure with intraoperative neuromonitoring. A “nerve alert” (neurologic signal change) occurred in 76.5%, was most frequent during glenoid exposure and graft insertion. Seven patients (20.6%) had a postoperative neurologic deficit, all of which involved the axillary nerve and resolved by 6 months. Total operative time and duration of the high-risk stages were significant predictors of postoperative neurologic deficit, while BMI and previous shoulder surgery were not.  
Freehill et al investigated changes in neurovascular anatomy following the Latarjet procedure in 4 cadaveric shoulder specimens. The axillary and musculocutaneous nerves both moved medial to their original position. The musculocutaneous nerve was found to consistently overlap the axillary nerve following the Latarjet procedure. The authors postulate that this may obscure future identification of either nerve and increase risk of neurologic injury in revision surgery. 
Shah et al analyzed the short-term complications of the Latarjet procedure. The overall complication rate was 25%, higher than previously reported in the
literature. Neurologic injury occurred in 10%, involving the axillary nerve (2), musculocutaneous nerve (2) and radial nerve (1). Recurrent instability occurred in 8% and 6% of patients developed a superficial wound infection.  
Figures A and B are the AP and axillary radiographs, respectively, demonstrating a coracoid autograft transfer stabilized with two 4.0mm non cannulated partially threaded screws with no evidence of hardware complication. Illustration A is from the AO foundation and demonstrates the neuroanatomy of the musculocutaneous and axillary nerves. The musculocutaneous nerve enters the coracobrachialis 3-8cm distal to the coracoid. The axillary nerve crosses the inferolateral border of the subscapularis 3-5mm medial to the myotendinous junction before continuing inferior to the glenohumeral joint and through the quadrangular space posteriorly.  
Incorrect Answers: 
Answer 2: While nerve traction during glenoid exposure is one of the most common mechanisms of injury, the neurologic deficit should first be observed for spontaneous recovery. An EMG should be obtained if recovery is not evident by 3-6 weeks postoperatively. 
Answer 3: Nerve entrapment is not a common mechanism of neurologic injury in the Latarjet procedure. 
Answer 4: While nerve traction is one of the most common mechanisms of injury and should be observed, neurologic injury during subscapularis split is far less common than during glenoid exposure or graft insertion. Answer 5: Nerve laceration is not a common mechanism of neurologic injury in the Latarjet procedure. 
OrthoCash 2020
116) A 22-year-old collegiate pitcher is having pain and decreased velocity with throwing. He is examined in the office and is diagnosed with Glenohumeral Internal Rotation Deficit (GIRD). He is prescribed a therapy regimen that involves internal rotation stretching at 90 degrees of forward flexion with the scapula stabilized. This will stretch which region of the shoulder joint? 
1. Anterior capsule 
2. Posterior capsule 
3. Antero-inferior capsule 
4. Rotator interval 
5. MGHL 
Corrent answer: 2 
The "sleeper stretch", or internal rotation stretch with the arm flexed 90- degrees and the scapula stabilized, stretches the posterior capsule preferentially as shown in Illustration A. Posterior capsular tightness is felt to be a cause of decreased internal rotation (GIRD) often seen in baseball pitchers. The other capsular areas are not significantly addressed by this stretch.  
Kibler et al provide a comprehensive discussion of shoulder rehabilitation, including principles of following a proximal-to-distal activation pathway, integrating various shoulder functions together in the rehab protocol, and emphasizing scapular control coupled with rotator cuff activation. 
OrthoCash 2020
117) A 45-year-old man has paralysis of his deltoid and rotator cuff as the result of a motorcycle injury. His neurologic injury has not improved in the 5 years since the injury, but he does have some use of his hand and wrist. What is the most appropriate treatment? 
1. Levator scapulae and rhomboid tendon transfer 
2. Total shoulder replacement 
3. Reverse total shoulder replacement 
4. Shoulder arthrodesis 
5. Latissimus dorsi transfer 
Corrent answer: 4 
A young patient with a flail shoulder but functional use of his hand is a good candidate for glenohumeral arthrodesis. The shoulder needs to be fused in a position of function, with mid-abduction and enough forward flexion and internal rotation so that his hand can reach his mouth. A typical arthrodesis position would be roughly 40 degrees of abduction, 35 degrees of foward elevation, and 20 degrees of internal rotation.  
Rühmann et al. reviewed 43 cases of shoulder arthrodesis performed for indications such as paralysis (32 [74%]), osteoarthritis or humeral head necrosis (15 [35%]), infection (7 [16%]), and persistent shoulder instability. The article has a surgical technique section that states, “In relation to the thorax, the following arm positions were selected for arthrodesis (range): abduction, 20° to 60°; anteversion, 20° to 40°; internal rotation 0° to 50°; and no external rotation.”  
Incorrect Answers:  
1. Eden-Lange procedure is used to treat chronic lateral scapular winging. It involves the lateral transfer of the levator scapulae, rhomboid major, and rhomboid minor to substitute for each part of the trapezius.  2. A total shoulder is contra-indicated in a patient without rotator cuff function. 3. A reverse TSA is contraindicated in patients without deltoid function.  5. Latissimus transfer is used to improve overhead motion in patients with irrepairable rotator cuff tears.  
 
118) Which of the following statements is true regarding the anatomical boundaries of the rotator interval in the shoulder? 
1. Superior border is defined by the biceps long head tendon 2. Inferior border is defined by anterior band of inferior glenohumeral ligament
3. Contains the axillary pouch which is a common site for intra-articular loose bodies 
4. Superior border is defined by anterior edge of suprapinatus tendon 5. Inferior border is defined by middle glenohumeral ligament 
Corrent answer: 4 
The rotator interval is the area between the anterior edge of the supraspinatus tendon superiorly, and the superior edge of the subscapularis tendon inferiorly. The medial border is comprised of the coracoid process and the lateral border is formed by the transverse humeral ligament.  
The rotator interval contains the biceps long head tendon, superior glenohumeral ligament, and coracohumeral ligaments. During shoulder arthroscopy, closure of the rotator interval can be used in patients with shoulder instability. Conversely, this area is often contracted in patients with adhesive capsulitis and may need to be released.  
Selecky et al. looked at cadaveric shoulder joints and the effects of arthroscopic thermal capsuloplasty on anterior and posterior glenohumeral translation. They found that closure or tightening of rotator interval was an effective way to significantly reduce anterior and posterior glenohumeral translation in vitro. 
Harryman et al. studied the role of the rotator interval in terms of shoulder passive motion and stability. Their results show that shoulder instability and occasional dislocation of the glenohumeral joint occurred inferiorly and posteriorly after sectioning of the rotator interval capsule. Meanwhile, imbrication of the rotator interval increased the resistance to inferior and posterior translation and provided more stability. 
Illustration A shows the anatomic boundaries and contents of the rotator interval.  
Incorrect answers: 
1- Long head of biceps is located within the rotator interval. 2- Anterior inferior glenohumeral ligament is located inferior to the rotator interval. 
3- Axillary pouch is located inferior to the rotator interval. 
5- Inferior edge is defined by superior border of subscapularis tendon.
 
119) A 35-year-old businessman complains of tingling and numbness in his fingers of both hands, mostly in the ring and small fingers, made worse with overhead activity. Neurologic exam and electromyography nerve conduction study is normal. His x-ray is shown in figure A. What is the most likely diagnosis? 
1. C5-6 cervical disk herniation 
2. C6-7 cervical disk herniation 
3. Bilateral cubital tunnel syndrome 
4. Bilateral radial tunnel syndrome 
5. Thoracic outlet syndrome 
Corrent answer: 5 
The thoracic outlet space is created by the clavicle, first rib, subclavius muscle, costoclavicular ligament, and anterior scalene muscle. It most often affects
subclavian artery, vein, and the lower trunk (C8 & T1) of the brachial plexus. The neurological exam may reveal sensory changes in the ring and little finger and intrinsic weakness. Radiographs here show cervical ribs, but could also show a Pancoast tumor or even be normal.  
Leffert reviewed thoracic outlet syndrome and cautioned that surgery be reserved for intolerable symptoms as surgical complications can be significant. 
Thompson provides a concise review stating that Thoracic outlet syndrome (TOS) represents a group of heterogeneous and potentially disabling upper extremity disorders that are caused by extrinsic compression of neurovascular structures between the first rib and clavicle. There are 3 distinct types of TOS, which are classified according to the principal anatomic structures involved and the clinical syndromes that result: neurogenic TOS, venous  TOS, and arterial TOS. 
 
120) What is the most common finding during surgery for traumatic anterior shoulder instability? 
1. Anterosuperior labral tear 
2. Anteroinferior labral tear 
3. Posterosuperior labral tear 
4. Posteroinferior labral tear 
5. Hill Sachs lesion 
Corrent answer: 2 
Hintermann et al conducted a prospective study to evaluate the arthroscopic findings of the unstable shoulder to provide insights in the causes and mechanisms of shoulder instability. 212 patients who had at least 1 documented shoulder dislocation. Of these 87% had anterior glenoid labral tears (the Bankart lesion), 79% had ventral capsule insufficiency, 68% had Hill-Sachs compression fractures, 55% had glenohumeral ligament insuffiency, 14% had complete rotator cuff tears, 12% had posterior glenoid labral tears, and 7% had SLAP tears. Similairly, In a study by Taylor et al, 63 of 67 (97%) had Bankart lesions, but none had rotator cuff tears. Thus, the right answer here is anterior labral tears, choice 2.  
OrthoCash 2020
121) A 63-year-old diabetic female complains of left shoulder pain and decreased range of motion 7 months after a fall onto her left side. On physical examination she has marked decrease in external rotation. A radiograph obtained earlier that day at her primary care office is displayed in Figure A. What is the next step in management? 
1. Physical therapy for adhesive capsulitis secondary to chronic 2-part humeral head fracture 
2. Proximal humeral arthroplasty 
3. Obtain further radiographic studies 
4. Open reduction and internal fixation of the chronic 2-part humeral head fracture 
5. Sling immobilization for 10-14 days then begin physical therapy for chronic 2-part humeral head fracture 
Corrent answer: 3 
Further radiographic studies are required including an axillary view. The humeral head resembles a "light bulb", indicating a possible posterior shoulder dislocation. Illustration A is an axillary view of this patient confirming chronic two-part fracture-dislocation of the anatomical neck of the humerus.  
The Level 4 study by Hawkins and Neer describe their experiences with 41 locked posterior dislocations of the shoulder with 50% having an associated fracture. Twenty of the dislocations were missed diagnoses by treating physicians and the average time from injury to diagnosis was 1 year. Hawkins and Neer conclude treatment is determined by the duration of time the
shoulder has been dislocated and the size of the humeral head defect, (reverse Hill-Sachs lesion).If the dislocation is less than 6 weeks old AND the defect involves less than 20% of the articular surface, then closed reduction should be attempted. If the dislocation is 6 weeks to 6 months old AND the defect involves 20 to 45% of the articular surface, transfer of the lesser tuberosity (McLaughlin procedure) should be done. If the glenoid is normal and the dislocation is more than 6 months old OR the defect involves more than 45% of the articular surface, or both, a hemiarthroplasty should be done. A hemiarthroplasty is indicated as treatment in this patient's case after obtaining full radiographs. It should be noted that adhesive capsulitis (option 1) does cause pain and loss of range of motion, especially with external rotation. 
 
122) An acute posterior shoulder dislocation should be suspected in a patient with pain and the shoulder locked in what position? 
1. Internal rotation 
2. External rotation 
3. Forward elevation 
4. Abduction 
5. Retraction 
Corrent answer: 1 
Patients with posterior shoulder dislocations are often overlooked. They present with the shoulder locked in internal rotation and adduction and lack external rotation. Orthogonal radiographs (anterior-posterior and axillary lateral) are absolutely necessary for proper diagnosis. Posterior dislocation patients often have reverse Bankart and reverse Hill-Sachs lesions. 
Ivkovic et al. reported a case of bilateral posterior dislocations successfully treated with hemiarthroplasty and osteochondral allograft. 
Longo et al performed a systematic review of 31 randomized controlled trials and concluded primary surgery is favorable in young adults engaged in highly demanding sports or job activities. However, there is lack of evidence to determine whether surgical or nonsurgical treatment is better for other categories of injury. 
Paul et al found 9 Level 4 articles in the evidence-based literature regarding posterior shoulder dislocation management. The size of the humeral head impression fracture is key for decision making. The algorithm in Illustration A provides treatment guidelines according to the size of the lesion. Nonoperative treatment can confer satisfactory results for older, lower demand patients or those with unstable epilepsy. However, surgical treatment is recommended to achieve good functional results in other patient populations. 
 
123) What technical error leads to scapular notching after reverse total shoulder arthroplasty? 
1. Superior placement of the glenoid component 
2. Retroverted placement of the glenoid component 
3. Inferior placement of the glenoid component 
4. Overtensioning of the soft tissue envelope 
5. Inferior tilt of the glenoid component 
Corrent answer: 1
Superior placement of the glenoid component during reverse shoulder arthroplasty can lead to scapular notching. This is the most common radiographic complication following the procedure. Illustration A shows this radiographic finding. 
Simovitch et al demonstrated that superior positioning of the glenoid component as well as superior tilt of the component with respect to the scapula were factors that led to scapular notching. Patients with this radiographic finding also had poorer clinical outcomes.  
Gutierrrez et al performed a biomechanical study which assessed the limits to range of motion with reverse shoulder implants. They found that inferiorly, adduction was limited by impingement on either the inferior scapular border or the glenoid. Superiorly, abduction was limited by impingement on the acromion. 
 
124) A 61-year-old laborer presents for total shoulder arthroplasty for primary osteoarthritis. What is his chance of having a concomitant full-thickness supraspinatus tear? 
1. less than 10% 
2. 10 to 20% 
3. 20 to 30%
4. 30 to 40% 
5. greater than 40% 
Corrent answer: 1 
Full-thickness supraspinatus tears have been historically rare in patients with primary shoulder osteoarthritis, with most studies showing a rate of < 10%.  
While numerous imaging studies in asymptomatic individuals have demonstrated a high prevelance of rotator cuff tears (30-55%) in individuals over the age of 60, this has not been the case with those patients presenting with primary shoulder osteoarthritis. This excludes patients with rotator cuff tear arthropathy, who by definition have a cuff tear and do not have "primary osteoarthritis".  
Norris and Iannotti noted 9% of their osteoarthritic patients presenting for arthroplasty had full-thickness supraspinatus tears. Edwards et al cited 42/514 (8%) of their osteoarthritic patients had a full-thickness supraspinatus tear. Please note that these numbers refer to full thickness tears of the supraspinatus in patents with primary osteoarthritis who are symptomatic as they are presenting for arthroplasty. 
 
125) In comparison to patients with osteoarthritis, patient with inflammatory arthritis undergoing shoulder arthroplasty are more likely to have? 
1. Large inferior humeral osteophyte 
2. Medialization of the glenohumeral joint line 
3. Posterior humeral head subluxation 
4. Sclerotic glenoid 
5. Posterior glenoid wear 
Corrent answer: 2 
Inflammatory arthritis (e.g. JRA, RA) of the shoulder characteristically demonstrates concentric glenoid erosion with medialization of the glenohumeral joint. As a result of the often severe gleniod erosion, glenoid resurfacing is not always feasible in these shoulder and some authors recommend hemiarthroplasty.  
The cited articles by Thomas et al and Jolles et al report good results of arthroplasty in patients with JRA. Eccentric wear (e.g. posterior glenoid wear,
posterior humeral head subluxation) may be seen in OA and in post capsulorrhaphy arthritis. Subchondral sclerosis is seen in OA with periarticular osteopenia is seen in inflammatory arthritis. A large inferior humeral osteophyte is commonly seen in advanced shoulder OA. 
 
126) A 31-year-old professional baseball pitcher has increased external rotation and a 30 degree deficit on internal rotation on his throwing shoulder compared to his non-dominant side. Motion analysis of the glenohumeral joint will show what abnormal movement of the humerus in relation to the glenoid during the cocking phase of throwing? 
1. Posterosuperior 
2. Posteroinferior 
3. Anteroinferior 
4. Anterosuperior 
5. Directly anterior 
Corrent answer: 1 
The clinical presentation is consistent for a glenohumeral internal rotation deficit (GIRD), typically caused by a posterior cuff and capsular contracture. Biomechanical studies have shown that the humerus is translated in a posterosuperior direction during the cocking phase of throwing in the setting of a posterior capsular contracture.  
GIRD is a common finding in pitchers and other throwing athletes. While it is not necessarily a painful condition, it has been shown to to alter the mechanics of the shoulder during throwing and may lead to injury of the superior labrum and articular side of the rotator cuff. Physical exam is significant for increased external rotation and loss of internal rotation compared to the contralateral shoulder. Tests for a SLAP tear may also be positive. In this condition, radiographs and MRI are often be normal.  
Grossman et al in a cadaveric study simulating GIRD by creating a posterior capsular contracture showed that the humerus moves in a posterosuperior direction during the cocking phase of throwing (external rotation in 90 degrees of abduction).  
Lintner et al report "therapy is directed at posterior capsular stretching to prevent loss of internal rotation and to protect the superior labrum and posterior rotator cuff". They also report that increased external rotation may
be attributable to increased humeral retroversion, while the internal rotation deficit is caused by soft tissue adaptations. 
Illustration A shows an algorithm for clinical reasoning in the examination of impingement related shoulder pain. 
 
127) What structure provides dynamic glenohumeral stability by compressing the humeral head against the glenoid? 
1. Superior glenohumeral ligament 
2. Middle glenohumeral ligament 
3. Teres major muscle 
4. Deltoid muscle 
5. Rotator cuff muscles 
Corrent answer: 5 
The rotator cuff is the main DYNAMIC stabilizer of the glenohumeral joint. It functions most at midrange motion, not at the extremes of range of motion. The superior glenohumeral ligament is a STATIC stabilizer and resists inferior translation at 0° degrees of abduction. The middle glenohumeral ligament is a STATIC stabilizer and resists anterior translation in the midrange of abduction (~45°) in ER. The teres major adducts and medially rotates arm and is not a significant stabilizer of the glenohumeral joint. The deltoid muscle primarily abducts the arm and is not the major stabilizer of the glenohumeral joint.  
Hirashima et al. used EMG to characterize the sequential activation of musculature during overarm throwing and postulate that the sequence is very effective for the generation of high force and energy in the trunk.
 
128) A professional baseball pitcher develops shoulder pain, weakness and lateral arm paresthesias in his throwing arm. Selected MRI images of the right shoulder are shown in Figures A and B. What is the diagnosis?
1. Parsonage-Turner syndrome 
2. Long thoracic neuritis 
3. Quadrilateral space syndrome 
4. Internal impingement 
5. External impingement 
Corrent answer: 3 
The patient has quadrilateral space syndrome with the resulting axillary nerve deficit.  
The axillary nerve passes through the quadrilateral space on its path to innervate the teres minor and deltoid and to provide sensation to the lateral arm. In throwers, the nerve can become irritated presumably from the repetitive motion leading to nerve dysfunction. This is usually self-limited.  
The McAdams paper describes good results in 4 overhead athletes treated with surgical release of the nerve following failure of nonoperative treatment.  
Figure A shows a T1-weighted MRI image of the posterior shoulder with fatty infiltration of the teres minor secondary to axillary nerve compression. Figure B shows a T1-weighted axial MRI image with fatty infiltration of the deltoid secondary to axillary nerve compression. 
 
129) A 65 year-old man has progressive debilitating pain and crepitus in his shoulder. Active forward elevation is 120 degrees and external rotation strength is normal. Radiograph and CT scan are shown in Figures A and B. Which treatment will likely give him the best outcome in 3 years.
1. Arthroscopic capsular release 
2. Humeral head arthroplasty with glenoid bone grafting followed by staged glenoid component implantation 
3. Hemiarthroplasty 
4. Reverse total shoulder replacement 
5. Total shoulder arthroplasty 
Corrent answer: 5
This patient has advanced glenohumeral humeral arthritis. The treatment that will lead to the best outcomes for this patient is a total shoulder arthroplasty (TSA). 
Use of TSA to treat glenohumeral arthritis has been associated with improved functional outcomes compared to hemiarthroplasty of the shoulder. Resurfacing of the glenoid should not be completed if there is insufficient glenoid bone stock. This patient has adequate bone stock, although there is evidence of posterior subluxation of the humeral head.  
Bryant et al. conducted a meta-analysis to compare outcomes between total shoulder arthroplasty (TSA) and hemiarthroplasty. The TSA demonstrated better functional outcomes (pain, UCLA shoulder score, forward elevation on exam) compared to hemiarthroplasty at a minimum of 2 years.  
Gartsman et al. prospectively evaluated outcome differences between TSA and hemiarthroplasty for shoulder osteoarthritis. At an average follow up of 35 months, the authors found that TSA provided greater pain relief, greater patient satisfaction, function and strength compared to hemiarthroplasty.  
Figure A shows an AP radiograph of the shoulder with evidence of osteoarthritis of the shoulder. Figure B shows an axial CT scan reconstruction of the same shoulder with posterior glenoid erosion.  
Incorrect Answers:  
Answer 1: Capsular release will not address the degenerative changes within the joint itself.  
Answer 2: While a hemiarthroplasty could be completed, evidence of posterior glenoid erosion is supportive of the use of a total shoulder arthroplasty. Bone grafting is indicated if there is greater than 20-25% posterior glenoid deficiency. Use of a total shoulder arthroplasty is associated with decreased need for revision surgery compared to hemiarthroplasty at short term follow up 
Answer 3: Hemiarthroplasty outcomes are not as good as with TSA Answer 4: Because this patient’s rotator cuff is intact, a reverse TSA is not indicated.  
 
130) A 40-year-old male was moving his furniture several days ago when he developed anterior forearm pain. On physical exam he is tender just distal to the antecubital fossa. He has decreased strength
on supination and elbow flexion when compared to the contralateral side. His MRI is shown in Figures A and B. His injury typically occurs in what portion of the tendon’s distal insertion? 
1. Proximal 
2. Distal 
3. Central 
4. Radial 
5. Ulnar 
Corrent answer: 4
The clinical presentation and MRI are consistent with a partial tear of the distal biceps tendon. Figure A shows inflammation at the site of the biceps insertion on the bicipital tuberosity with some attached tendon fibers remaining. Figure 
B shows abnormally increased signal intensity and an increased diameter of the distal biceps tendon (arrow) compatible with a partial tear. Avulsive marrow edema is present within the bicipital tuberosity of the radius (arrowheads) and distension of the bicipitoradial bursa is present (short arrow). Partial tears of the distal biceps tendon are rare and may be frequently misdiagnosed. It typically occurs in active, middle-aged males, and only a small number have been reported in the literature.  
Davis et al published a case report showing that the tear is degenerative in nature, and is located along the radial border of the bicipital tuberosity where spurring has occurred.  
This was confirmed by Kelly et al who found the same partial tear pattern and described a surgical technique to repair it through a single incision posterior approach. The illustration in Figure A explains the footprint of the distal biceps. 
 
131) All of the following protect the elbow from valgus loads during the throwing cycle EXCEPT? 
1. Flexor-pronator muscle contraction 
2. Reduced fastball velocity 
3. Increased glenohumeral internal rotation torque 
4. Forearm pronation 
5. Scapular protraction/retraction 
Corrent answer: 3
During a thrower's kinetic chain, increased shoulder internal rotation torque contributes to increased valgus elbow loads. Marshall et al described the importance of proximal to distal control of the upper extremity in producing angular and racket velocity and theorized that longitudinal rotations should be considered during injury prevention programs. Davis et al demonstrated that correct pitching mechanics offered the most efficient throwing velocity for a given shoulder internal rotation torque and elbow valgus load. Thus scapular dynamic control, correct pitching mechanics, pronation of the forearm with dynamic flexor-pronator muscle contraction protects the elbow from valgus loads. 
 
132) A football player subluxates his shoulder while blocking with his arm forward flexed and internally rotated. The “Jerk” test is positive. What is his most likely pathology? 
1. Bankart lesion 
2. Kim lesion 
3. Hill-Sachs lesion 
4. Glenohumeral internal rotation deficit 
5. Acromioclavicular separation 
Corrent answer: 2 
This football player likely suffered a posterior inferior labral tear, also known as a Kim lesion, or a Reverse Bankart lesion. The Jerk test is where a posterior force is applied along the axis of the humerus with the arm in forward flexion and internal rotation. This will cause the humeral head to subluxate posteriorly out of the glenoid socket. As the arm is brought into extension, a clunk will be felt as the humerus reduces into the glenoid cavity. A Hill-Sachs lesion is a bony humeral impaction lesion from an anterior dislocation, glenohumeral internal rotation deficit (GIRD) is found in throwers who have a tight posterior inferior capsule and lack internal rotation, an acromioclavicular separation has pain over the AC joint and may have a positive piano key sign when the clavicle “pops” up with arm depression.  
The review by Millett describes the history, exam, and radiologic findings of posterior shoulder instability.  
The Kim paper describes the injury which bears his name. 
OrthoCash 2020
133) Figure A shows the characteristic microscopic findings of lateral epicondylitis. Which of the following is the most appropriate term to describe the abnormal finding in the region marked with the two asterisks? 
1. Osteoblastic rimming 
2. Reactive hyperemia 
3. Localized hemorrhage with neutrophils proliferation 
4. Angiofibroblastic dysplasia 
5. Cystic degeneration with fatty infiltration 
Corrent answer: 4 
The region marked by the asterix in Figure A demonstrates angiofibroblastic dysplasia, which is a term used to collectively describe the microscopic changes typically seen with lateral epicondylitis. These pathologic changes include fibroblast hypertrophy, disorganized collagen, and vascular hyperplasia. 
Lateral epicondylitis or "tennis elbow" is a common condition resulting from repetitive wrist and elbow extension. It is most commonly located at the origin of the ECRB tendon with pain just distal to the lateral epicondyle.  
Kraushaar et al describe the histology as "immature fibroblastic and vascular infiltration of the origin of the extensor carpi radialis brevis" and this description has been termed "angiofibroblastic dysplasia" (answer 4).  
Nirschl et al reviewed nearly 1200 patients with lateral epicondylitis and only
88 required operative care with 97% reporting improvement and 85% returning to rigorous sport after surgery. 
 
134) Which of the following structures shares the same origin site as the tendon that undergoes angiofibroplastic hyperplasia during the pathogenesis of tennis elbow? 
1. Brachioradialis 
2. Anconeus 
3. Annular ligament 
4. Flexor carpi ulnaris 
5. Palmaris longus 
Corrent answer: 2 
Lateral epidondylitis is classically thought to be caused by histopathologic angiofibroblastic hyperplasia at the origin of the extensor carpi radialis brevis. ECRB originates from the common extensor wad, that also includes ECRL, ED, ECU. The anconeus shares the same attachment site at the lateral epicondyle as the ECRB (as shown in Illustration A). 
The classic Level 4 study by Nirschl reviewed 1,213 patients with tennis elbow of which 88 elbows underwent surgery. Immature fibroblastic and vascular infiltration of the origin of the ECRB was found, excised, and tendon repaired by Nirschl with an improvement rate of 97%. 
OrthoCash 2020
135) A 10-year-old little league pitcher has the triad of medial elbow pain in his throwing arm, decreased throwing effectiveness, and decreased throwing distance. What is the pathogenesis of the condition that is most likely to be occuring in this patient? 
1. Acute fragmentation of the entire capitellar ossific nucleus 2. Rupture of the anterior band of the anterior bundle of the ulnar collateral ligament 
3. Repetitive contraction of the flexor-pronator mass stresses the chondro osseous origin, leading to apophysitis 
4. Microtraumatic vascular insufficiency of the capitellum from chronic compressive and rotatory forces 
5. Repetitive triceps contraction during extension 
Corrent answer: 3 
This adolescent pitcher's symptoms characterize Little Leaguer's elbow. Little Leaguer' elbow results from repetitive valgus stresses and tension overload of the medial structures.  
Chen et al. presents Level 5 evidence demonstrating that repetitive contraction of the flexor-pronator mass stresses the chondro-osseous origin at the medial epicondyle, leading to inflammation and subsequent apophysitis. Radiographic changes may range from normal to irregular ossification of the medial epicondylar apophysis, followed by accelerated growth, marked by apophyseal enlargement, separation, and eventually fragmentation.  
Limpisvasti et al presents Level 5 evidence that treatment of Little Leaguers elbow is initially conservative with rest followed by a gradual return to activities. 
Illustration A shows an MRI scan representing medial epicondyle apophysitis typical of Little Leaguer's Elbow.
 
136) Which patient would be ideal for an open shoulder reduction and glenoid bone augmentation? 
1. 25-year-old with first time acute traumatic dislocation 
2. 78-year-old with a rotator cuff tear arthropathy with superior escape 3. 24-year-old with chronic dislocation and large engaging Hill-Sachs lesion 4. 30-year-old with an acute bony Bankart fracture-dislocation 5. 27-year-old with a chronic anterior dislocation and inverted pear-shaped glenoid 
Corrent answer: 5 
Open reduction and glenoid bone augmentation with graft or coracoid transfer is ideal for chronic dislocations with anterior glenoid deficiency (inverted pear shaped glenoid) without significant Hill-Sachs (<20% of humeral head arc impaction).  
The algorithm to treat chronic dislocation was reviewed by Shahajpal. When the glenoid defect is greater than 20-30% then bony augmentation is indicated. The humeral head defect should be addressed if engaging or 20- 40% head loss, and hemiarthroplasty should be considered if >40% of the
head is involved.  
The Beran paper reviews management of glenoid bone defects. 
 
137) The pathology seen in Figure A is most likely to result from trauma that occurred with the shoulder in which of the following positions? 
1. Adduction, internal rotation 
2. Adduction, external rotation 
3. Abduction, external rotation 
4. Extension, internal rotation 
5. Axial traction in adduction 
Corrent answer: 3 
Figure A is an MRI arthrogram showing an anterior labral injury and a Hill Sachs defect which are both characteristic of anterior shoulder instability. The classic position of anterior shoulder instability is abduction and external rotation. Posterior instability is associated with trauma in the position of adduction and internal rotation.  
Sanders et al suggests that magnetic resonance arthrography is the study of
choice for evaluation of soft tissue lesions associated with shoulder instability. 
Illustrations A & B also show axial MRI arthrogram images. Note how A shows the labral tear at the level of the rotator interval, so the subscapularis tendon is not seen well. Image B is further inferior and clearly shows that the subscapularis is intact. 
 
138) Mineralization of the posterior-inferior glenoid has been implicated as a possible source of pain in which athletic population?
1. football players 
2. swimmers 
3. basketball players 
4. rowers 
5. baseball pitchers 
Corrent answer: 5 
The Bennett's lesion is mineralization of the posterior-inferior glenoid observed in overhead athletes. it is felt to be a traction spur of the posterior inferior glenohumeral capsule which is repetitively stressed during the deceleration and follow-through phases of the throwing cycle. 
Wright and Paletta documented 22% of major league pitchers in their series had radiographic evidence of a Bennett's lesion, although none required surgical treatment. 
Illustration A is an axillary radiograph showing a posterior glenoid radiodensity consistent with a Bennett's lesion. 
 
139) An 18-year-old football player sustains an anterior shoulder dislocation that is reduced on the field. When he presents to the office complaining of posterior pain, you suspect a Hill-Sachs defect. Which of the following is the best radiographic view for identifying a Hill Sachs defect?
1. Figure A 
2. Figure B 
3. Figure C
4. Figure D 
5. Figure E 
Corrent answer: 3 
The Stryker notch view (Figure C) is best for identifying a Hill-Sachs defect. It is obtained by positioning the patient supine with the arm flexed toward the ceiling, flexed at the elbow, and the patient’s hand placed on top of the head and the xray beam is directed anteroposteriorly, with 10° of cephalic angulation. (Illustration A) 
The “true” AP or Grashey shoulder view (Figure A) is obtained by tilting the x ray beam in the plane of the scapula, approximately 45° laterally off the AP shoulder view to obtain an "end on" view of the glenohumeral joint. The scapular “Y” view (Figure B) is obtained by tilting the x-ray beam along the plane of the scapula, approximately 60° relative to the AP view.  
The article by Sanders et al states the Stryker view is poor for detecting Bankart lesions and subluxation, but good for finding Hill-Sachs defects. The Zanca view (Figure D) is utilized to delineate Acromioclavicular (AC) joint anatomy and is a shoulder AP view focusing on the AC joint with a 10 degree cephalic tilt. An axillary view (Figure E) is the best option for detecting an anterior shoulder dislocation. The radiograph beam is projected in a distal-to proximal direction through the shoulder with the arm in abduction and provides a view tangential to the glenohumeral joint.  
Sanders et al lists 10 different variations of the axillary view with modifications to help with patient comfort (Velpeau view eliminates need to abduct arm) or delineate specific anatomic features (West Point view for Bony Bankarts). The 2 review articles by Sanders et al are excellent primers for obtaining and interpreting shoulder imaging.
 
140) A 16-year-old swimmer has pain and weakness in her dominant shoulder with overhead use. Her physical examination demonstrates a +2 anterior and posterior load and shift test. There is 1.5cm of sulcus sign evident with the arm at adduction and 30 degrees of external rotation. Her radiographs are normal. What is the most appropriate next step in management? 
1. Arthroscopic anterior and posterior labral repair 
2. Arthroscopic anterior and posterior labral repair with rotator interval closure 3. Home stretching program with emphasis on posterior capsular stretching 4. Dynamic stabilization therapy 
5. Sport specific bracing 
Corrent answer: 4 
Multidirectional instability (MDI) is defined by symptomatic global laxity of the glenohumeral joint with increased translation in multiple planes (ie. anterior, inferior, and posterior directions). It is usually an atraumatic patholaxity and should be initially treated by physical therapy to strengthen and retrain the proprioreceptive response of the muscular stabilizers of the shoulder.  
Lee et al performed a cadaveric study and each rotator cuff muscle contributed to dynamic stabilization of the glenohumeral joint in various arm positions.  
Schenk and Brems reviewed MDI and noted that shoulder laxity was typically bilateral but the symptomatic shoulder usually correlates with loss of strength, poor neuromotor coordination of the rotator cuff and scapular stabilizing muscles, and defective proprioceptive responses. If a 6-month trial of
nonoperative management fails, the patient is a candidate for surgical stabilization. 
 
141) A 50-year-old man sustains a left shoulder injury after falling from a motorcycle. A physical examination test to examine for this shoulder injury is found in Figure A. What is the most likely diagnosis? 
1. SLAP tear 
2. Supraspinatus tear 
3. Infraspinatus tear 
4. Teres minor tear 
5. Subscapularis tear 
Corrent answer: 5 
The concept tested in this question is whether you know what physical exam finding is associated with a subscapularis tear.  
The key physical exam findings are positive Belly-press and Lift-off maneuvers, as well as weakness in internal rotation and increased passive external rotation. The MRI confirms the diagnosis with discontinuity of the subscapularis and the long-head of the biceps not located in the bicipital groove. 
Sanders and Miller present a Level 5 review article detailing the correlation between MR images and clinic examinations for shoulder pathology. Part 2 of their 2-part series is also a valuable resource and discusses pathology of the knee. 
Tennent et al. present a Level 5 review article discussing the key physical examination pearl for testing the rotator cuff. Part 2 of 2-part series discusses instability and SLAP lesion tests.  
Figure A depicts an abnormal belly-press examination whereby the wrist flexes, and the elbow will fall posteriorly as the patient recruits the posterior deltoid to compensate for lack of the subscapularis. Illustration A demonstrates a normal belly-press examination. Illustration B demonstrates a subscapularis tear and associated long head of the biceps dislocation.
 
142) Static progressive turnbuckle splinting is most appropriate for which of the following patients? 
1. 3 months after ORIF of a distal humerus fracture with a flexion arc of 45° to 100° with no further improvement with physical therapy 
2. 4 weeks after nonoperative treatment of a displaced radial head fracture with block to supination 
3. 1 week after simple elbow dislocation with flexion arc of 10° to 140° 4. Presence of extensive heterotopic ossification after a complex elbow dislocation with associated ankylosis of the joint 
5. Immediatly after elbow arthroscopy for loose body removal and debridement 
Corrent answer: 1 
Static progressive splinting is useful treatment for certain patients with post traumatic elbow stiffness. Generalized accepted indications are flexion contractures greater than 30 degrees, or flexion less than 130 degrees after a failed trial of physical therapy.  
Gelinas et al treated 22 patients with an elbow contracture using a static progressive turnbuckle splint for a mean of 4.5 months. Eleven patients gained a 'functional arc of movement,' defined as at least 30 degrees to 130 degrees, and only 3 patients showed no improvement at all. 
Doornberg et al treated 29 consecutive patients with elbow stiffness after trauma (flexion contracture greater than 30 degrees or flexion less than 130 degrees) who had failed therapy. The flexion arc improved from 71 degrees before splinting to 112 degrees after splinting. Operative treatment of stiffness was avoided in most patients. 
Incorrect Answers: 
2-Progressive turnbuckle splinting is typically not indicated for mechanical blocks to motion, especially in the supination/pronation plane. 3-A functionally physiologic flexion arc of 10-140 degrees is not an indication for turnbuckle splinting, as it will likely not be improved with this treatment.  4-An ankylosed joint will not benefit from progressive turnbuckle splinting. 5-Turnbuckle splinting is not typically prescribed after elbow arthroscopy for loose body removal.  
An example of a turnbuckle splint is shown in Illustration A.
 
143) A 62-year-old man undergoes a total shoulder arthroplasty for osteoarthritis. He accidently uses his operative arm to rise from a chair 3 weeks after surgery and thereafter complains of anterior shoulder pain. Radiographs are significant for anterior dislocation of the prosthesis. What is the most likely mechanism for this complication? 
1. long head biceps rupture 
2. supraspinatus rupture 
3. subscapularis rupture 
4. infraspinatus rupture 
5. labral tear 
Corrent answer: 3 
The primary restriction after total shoulder arthroplasty (TSA) is passive external rotation, as well as active internal rotation, to protect the subscapularis repair. This patient fired his subscapularis rising from the chair.  
According to Wirth and Rockwood, rupture of the subscapularis was seen in all cases of anterior dislocation following TSA. 
 
144) A 35-year-old male injured his right shoulder while playing basketball. He presents emergently with significant pain and his shoulder abducted at 140 degree. He is unable to lower his arm. 
Radiographs will most likely show that his humeral head has dislocated in what direction? 
1. Anterior 
2. Posterior 
3. Superior
4. Inferior 
5. Lateral 
Corrent answer: 4 
This patient has suffered an inferior shoulder dislocation (luxatio erecta). This is a rare type of shoulder dislocation, representing only about 0.5% of all shoulder dislocations. Anterior and posterior dislocations are much more common but don't present with a shoulder fixed in abduction or flexion such as this case.  
The cited articles by Seweckee and Fung report that this condition can be diagnosed by physical exam alone due to its unique presentation, but radiographs should be obtained to evaluate for concomitant injuries.  
Since Pubmed does not have abstracts on these articles, the Fung article can be accessed at http://www.amjorthopedics.com/pdfs/037050097e.pdf 
Illustration A demonstrates a right shoulder radiograph of an inferior shoulder dislocation with abduction of the arm present. Illustration B displays a clinical image of a patient with an inferior shoulder dislocation.
 
145) In valgus extension overload of the elbow, which letter in Figure A corresponds to the typical location of osteophytes formation?
1. A 
2. B 
3. C 
4. D 
5. E 
Corrent answer: 4 
Valgus extension overload syndrome of the elbow occurs in throwing athletes. The mechanism is thought to be valgus stress on the elbow during acceleration, especially with insufficiency of the medial ulnar collateral ligament, as well as extension impaction forces during decceleration. Over time, the continuous impaction of the posterior-medial olecranon in the olecranon fossa can lead to chondromalacia and osteophyte formation.  
Wilson et al. described this ostoephyte formation in pitchers which were successfully treated with osteophyte removal. This can be done either arthroscopically or with a limited open arthrotomy. 
Illustration A depicts the biomechanics responsible for elbow valgus extension overload syndrome.
 
146) A 58-year-old right-hand dominant accountant falls off a bicycle 4 days ago and injured his left non-dominant shoulder. A radiograph is shown in Figure A. The axillary radiograph shows no antero-posterior translation. What is the most appropriate next step in treatment? 
1. Coracoclavicular ligament reconstruction 
2. Acromioclavicular capsular reconstruction 
3. Sling and early ROM exercises 
4. Arthroscopic distal clavicle excision 
5. Weaver-Dunn procedure 
Corrent answer: 3
The radiograph demonstrates an acromio-clavicular (AC) separation. Whether this is graded as either a Type II or III, non-operative treatment is recommended for this patient. A Type II shows partial separation at the AC and no widening of the coracoclavicular (CC) space, which can be difficult to assess without an image of the uninjured side. Type II is generally treated non-operatively. A Type III has generally has 100% displacement at the AC and widening of the CC space (widened 20-100%). While the treatment of type III may be controversial in athletes or laborers, the treatment of this patient (non-dominant shoulder, non-laborer and non-athlete) should be non operative initially.  
Schlegel et al prospectively followed 25 patients treated conservatively with grade III AC separations and showed that at 1 year there was no limitation of shoulder motion and no difference between sides in rotational shoulder muscle strength. 
 
147) During diagnostic arthroscopic evaluation of a patient's shoulder, you identify a thickened portion of the coracohumeral ligament, near its avascular zone, running perpendicular to the supraspinatous tendon. The structure is identified in Figure A with black arrows. What is the name for this structure? 
1. Middle glenohumeral ligament 
2. Rotator interval 
3. Coracoid process 
4. Rotator cable 
5. Rotator crescent 
Corrent answer: 4 
The black arrows in Figure A identify the rotator cable, while the asterisk identifies the rotator crescent.
The rotator crescent and rotator cable are two anatomic structures closely associated with the rotator cuff that form the shoulder's "suspension bridge." The rotator crescent is a thin, crescent-shaped sheet of rotator cuff comprising the distal portions of the supraspinatus and infraspinatus insertions. The crescent is bounded at its proximal margin by a thick bundle of fibers called the rotator cable, and thickened portion of supraspinatus. This cable-crescent spans the insertions of supraspinatus and infraspinatus tendons. The rotator cable is over 2.5 times thicker than the rotator crescent as measured by digital micrometer.  
Clark and Harryman describe the complex anatomy of the rotator cuff including the "suspension bridge" of the rotator cuff and crescent in their anatomical sectioning of 32 grossly intact cuffs.  
Burkhart and Lo review the anatomy, biomechanics, surgical techniques, and outcomes of the arthroscopic rotator cuff repair. They emphasize that achieving a biomechanically stable construct is critical to biologic healing, and discuss the different variables to obtain such a construct. 
Illustration A shows an arthroscopic photo annotated with the rotator cable (arrows), rotator crescent (asterisk), biceps tendon (BT), and humeral head (HH). Illustration B illustrates the relationship between the rotator crescent (CRES) and rotator cable (Label C). Illustration C shows a coronal cut of the supraspinatus, and how a thickened portion contributes to the rotator cable. 
Incorrect Answers: 
1) Middle glenohumeral ligament - anterior shoulder capsular thickening which provides resistance to anterior translation with the arm in 45 degrees of abduction 
2) Rotator interval - the "space" between the anterior leading edge of the supraspinatus tendon and the superior border of the subscapularis tendons. 3) Coracoid process - the "light-house" of the shoulder, a bone projection off the anterior portion of the scapula, just medial to the glenoid. 5) Rotator crescent - thin sheet of rotator cuff comprising the distal portion of the supraspinatus and infraspinatus tendon insertions.
 
148) A 37-year-old severe asthmatic has been taking daily corticosteroids for twenty years and now reports 4 months of worsening left shoulder pain. He is unable to complete a full day of work due to the pain. A radiograph is provided in Figure A. Which of the following describes the pathogenesis behind this disease process?
1. Cell-mediated immune response inciting synovial hypertrophy and mononuclear destruction of cartilage 
2. Humoral immune response following a systemic infection in an HLA-B27 positive individual 
3. Hyperuricemia induced crystalline deposition within the synovial fluid 4. Cellular death of the subchondral bone following an interruption in the vascular supply 
5. Bacterial seeding of the joint inducing polymorphonuclear cell destruction of the cartilage 
Corrent answer: 4 
The radiograph demonstrates AVN of the humeral head and early collapse of the articular surface. Patients with radiographic evidence of proximal humerus AVN should also have hip radiographs performed as part of their evaluation. 
Cruess reviews 95 patients with steroid-induced AVN of the humeral head reporting success with both conservative and arthroplasty treatment.  
L'insalata et al reviews 65 shoulders with AVN of the humeral head reporting mixed results with 35 shoulders requiring arthroplasty after failure of conservative therapies. Surgical drilling and decompression did not alter the progression of disease. 
Incorrect Answers:
Answers 1, 2, 3, and 5 describe rheumatoid arthritis, Reiter's Syndrome, gout, and septic arthritis respectively. 
 
149) Rotator cuff tears (full thickness and partial thickness) in asymptomatic individuals are seen on MRI or ultrasound in what percentage of patients over the age of 60? 
1. 0-5% 
2. 5-30% 
3. 30-55% 
4. 55-80% 
5. 80-100% 
Corrent answer: 3 
The prevalence of asymptomatic rotator cuff tears diagnosed with MRI or ultrasound is quite high, with most studies noting tears in 30-54% of patients over the age of 60. The paper by Tempelhof et al utilized ultrasound and found tears in 30% of patients over 60. The study by Sher et al noted an overall prevalence of 54% on MRI in patients older than 60 (28% had full thickness and 26% with partial thickness). In both studies the frequency of tears increased substantially with increasing age. 
 
150) A 26-year-old outfielder undergoes arthroscopic repair of a right shoulder type 2 SLAP tear with two labral anchors in the 11 and 1 o’clock positions. Postoperative rehabilitation for this SLAP repair should include: 
1. Immediate full active range of motion that simulates sport-specific activities 2. Full-time sling wear with no active nor passive motion for at least 6 weeks until labral tissues heal 
3. Rotator cuff strengthening by post-operative week two to prevent disuse atrophy and shoulder instability 
4. Limited passive motion for 4 weeks then progressive active motion until 8 weeks followed by sport specific strengthening until at least 12 to 16 weeks postoperatively 
5. Eccentric open chain biceps contraction exercises beginning at postoperative week 2 to retrain the biceps muscle and stimulate SLAP healing at the biceps anchor on the glenoid
Corrent answer: 4 
The most common type 2 SLAP repair rehabilitation protocols focus on limited passive range of motion followed by regaining active motion, then finally strengthening and sport specific activities. Specific protocols vary by surgeon, concomitant injuries, and intra-operative findings. The initial passive motion is often limited from 0 to 90 degrees of forward elevation during the first 4 weeks. This helps prevent stiffness, but also avoids stressing the repair site at the extremes of motion. Active motion is then initiated in a limited arc of motion to re-strengthen the shoulder. Finally, sport specific activities are progressively trained from 8 to 16 weeks.  
The Ellenbecker OKU reference reviews shoulder rehab and the Ellenbecker et al reference describes therapy focused on early passive motion with progressive active motion for rotator cuff repairs. They describe successful return of near full ROM and strength by 12 weeks postoperatively. The Manske et al article reviews the therapy for all SLAP injuries treated both operatively and non-operatively

related links ORTHOPEDIC MCQS OB 20 SHOULDER AND ELBOW 1

ORTHOPEDIC MCQS OB 20 SHOULDER AND ELBOW 2

ORTHOPEDIC MCQS OB 20 SHOULDER AND ELBOW 3

ORTHOPEDIC MCQS OB 20 SHOULDER AND ELBOW4

ORTHOPEDIC MCQS OB 20 SHOULDER AND ELBOW 5

ORTHOPEDIC MCQS OB 20 SHOULDER AND ELBOW 6

FOR ALL MCQS CLICK THE LINK ORTHO MCQ BANK